Variance of the sum of random independent variables

Click For Summary
To compute the variance of the sum of independent random variables, use the formula σ²_y = Nσ²_x, where σ²_x is the variance of the individual random variables x_i and N is the number of variables being summed. The discussion highlights the importance of understanding the variance for the case when N equals 2, which serves as a foundation for generalizing to larger N. A misunderstanding of the variance formula for y was noted, as it incorrectly assumes a uniform distribution. The correct approach emphasizes the independence of the x_i variables in calculating the variance of their sum. Understanding these principles is crucial for accurately determining the variance of sums of random variables.
dipole
Messages
553
Reaction score
151

Homework Statement



let x_{i} be a random variable, and let y_{j} = \sum x_{i}.

The variance of the random distribution of the x_{i}'s is known, and each y is the sum of an equal amount of x_{i}'s, say N of them.

How do I compute the variance of y in terms of \sigma^2_{x} and N?

Homework Equations



\sigma^2_{y} = \sum\frac{(y - \mu_{y})^2}{M}
 
Physics news on Phys.org
Also forgot to mention that I already know that \mu_{y} = N\mu_{x}.
 
dipole said:

Homework Statement



let x_{i} be a random variable, and let y_{j} = \sum x_{i}.

The variance of the random distribution of the x_{i}'s is known, and each y is the sum of an equal amount of x_{i}'s, say N of them.

How do I compute the variance of y in terms of \sigma^2_{x} and N?

Homework Equations



\sigma^2_{y} = \sum\frac{(y - \mu_{y})^2}{M}

I don't understand your formula for \sigma^2_{y}, which would be false for every probability distribution I can think of. (It assumes Y is a uniformly-distributed random variable taking M distinct values.)

Start with the simple case N=2: Y = X_1 + X_2, where X_1,\; X_2 are independent. Once you have done that case, the general case follows almost immediately.

RGV
 
Question: A clock's minute hand has length 4 and its hour hand has length 3. What is the distance between the tips at the moment when it is increasing most rapidly?(Putnam Exam Question) Answer: Making assumption that both the hands moves at constant angular velocities, the answer is ## \sqrt{7} .## But don't you think this assumption is somewhat doubtful and wrong?

Similar threads

Replies
7
Views
2K
Replies
0
Views
837
  • · Replies 2 ·
Replies
2
Views
1K
  • · Replies 8 ·
Replies
8
Views
2K
Replies
1
Views
1K
  • · Replies 5 ·
Replies
5
Views
905
  • · Replies 5 ·
Replies
5
Views
2K
  • · Replies 2 ·
Replies
2
Views
2K
  • · Replies 1 ·
Replies
1
Views
2K
  • · Replies 2 ·
Replies
2
Views
2K